随机过程标记的主动问题-数学堆栈交换 math.stackexchange.com最新30条 2024-06-23T19:06:39Z https://math.stackexchange.com/feeds/tag?标记名=随机-流程&;sort=最新 https://creativecommons.org/licenses/by-sa/4.0/rdf https://math.stackexchange.com/q/4933797网址 1 “启发式”与严谨的伊藤引理 简·斯图勒 https://math.stackexchange.com/users/503335 2024-06-17T14:40:12Z年 2024-06-23T18:13:42 Z <p>假设<span class=“math-container”>$X_t$</span>是一个标准的Brownian运动,而$t$</sspan>是时间变量,我学会了使用以下结果导出函数的Ito引理:</p><p>对于任何有限的<span class=“math-container”>$\delta t&gt;0$</span>和标准布朗运动$X_t$</sspan>,我们可以认为:</p><p><span class=“math-container”>\开始{align*}\标记{1}\增量X_t:=X(\delta t)\stackrel{d}{=}\sqrt{\delta t}X(1)\结束{align*}</p><p><span class=“math-container”>\开始{align*}\标记{2}\mathbb{E}[\delta X^2]=\mathbb}[\delta t X(1)^2]=\ delta t\结束{align*}</p><p>作为$\delta t到0$</span>,我们可以证明(为了便于表示):</p><p><span class=“math-container”>\开始{align*}\标记{3}V(δX^2)=V(δTX^2]=δt^2 V(X^2\结束{align*}</p><p>(最后的结果是正确的,因为使用正态分布的矩生成函数,$delta t^2到0$</span>为$delta t到0$</span>)</p>(第页)<p>我们现在可以定义$\delta F:=F(X_0+\delta X_t,t_0+\ delta t)-F(X_0,t_0)$</span>并使用泰勒展开式,如下所示:</p><p><span class=“math container”>$$\ delta F=\frac{\partial F}{\partial t}\ delta t+\frac{\partial F}{\partial t}\ delta X_t+0.5\frac{\partial ^2 F}{\partial X_t^2}\ delta X_t^2+$$</span></p><p>所有高阶项都变为零,即$\delta t到0$</span>,使用结果(1)、(2)和(3),我们可以证明$\deltaX_t^2到delta t$</sspan></p>(第页)<p>使用符号<span class=“math-container”>$\delta t\to 0\leftrightarrow\delta t\ to dt$,我们最终得到:</p><p>$$dF=\left(\frac{\partial F}{\partic t}+0.5\frac}\partial^2F}{\ partial X_t^2}\right)d t+\frac\\partial F}{\protial t}d X_t$$</p><p><strong>问题</strong>:上述内容通常被描述为;启发式“;推导了$F(X_t,t)$</span>的Ito公式。会是什么;非启发式;(数学严谨?)推导版本?主要区别是什么/我如何改进;启发式;上述论点使其更加严格</p>(第页) https://math.stackexchange.com/q/4936083 0 如果$S$和$T$是停止时间,那么$(S\leq T)\in\mathcal是真的吗{F} _T(_T)$? 没有人 https://math.stackexchange.com/users/875120 2024-06-22T08:43:11Z年 2024-06-23T17:19:11Z <p>假设$S$和$T$是关于连续过滤的停止时间{F} _(t))_{t\geq0}$</span>。<span class=“math-container”>$(S\leq T)\in\mathcal是真的吗{F} _T(_T)美元</span></p>(第页)<p>如果假设过滤是离散的而不是连续的,那么通过考虑所有<span class=“math-container”>$k\in\mathbb{N}$</span>,<span class=”math-continer“>$(S\leq-T)\cap(T=k)=(S\leq-k)\ cap(T=k)\ in\mathcal,很容易证明答案确实是肯定的{F} k(_k)美元</span>。然而在连续的情况下,这种推理似乎不起作用,因为它会导致将$(S\leq t)\cap{F} _(t)美元</span></p>(第页) https://math.stackexchange.com/q/1351556 14 为什么可预测的随机过程称为“可预测”? 0xbadf00d(错误00d) https://math.stackexchange.com/users/47771 2015年7月6日T17:01:39Z 2024-06-23T17:02:25Z <p>让</p><ul><li>$(\Omega,\mathcal A,\operatorname P)$是概率空间</li><li>$I$是一个索引集</li><li>$\mathbb F=(\mathcal F)_{t\inI}$是对$(\Omega,\mathcar a)的过滤$</li><li>$X=(X_t)_{t\inI}$是$(\Omega,\mathcal a,\operatorname P)上的随机过程$</li></ul><小时><p>如果$I=\mathbb{N} 0$,则$X$称为$\mathbb F$-<strong>可预测</strong>$:\Leftrightarrow$$X_0$是一个常量,$$X_n\text{is}\mathcal F_{n-1}\text{-measuble}\;\;\文本{for-all}n\in\mathbb n\;。\标记{1}$$如果人们认为$\mathcal F_n$是$X$在$n$之前的已知信息,$(1)$意味着在$n-1$时,我们已经知道$X_n$的行为</p>(第页)<小时><p>如果$I=[0,\infty)$,则$X$称为$\mathbb F$-<strong>可预测</strong>$:\Leftrightarrow$X$相对于$$\sigma\left(\left\{left\}0\right\}\ times F:F\in\mathcal F_0\right\}\cup\bigcup_{0\le s&lt;t}\left\{(s,t]\ times F:F\in\fathcal F_s\right\\}\right)\;.$$让$t&gt;0$. 现在,直觉更难找到。</em>$t$之前没有单独的数字$s$<em>。很明显,$s&lt;t$并不意味着$X_t$的$\mathcal F_s$可测量性</p>(第页)<p>那么,什么是直觉?也许我们可以证明$X_t$相对于$$\bigcup_{0\les&lt;t}\mathcal F_s\tag{2}\;是可测量的如果这是真的,并且有人能提供证据,我将非常高兴。ic</p> https://math.stackexchange.com/q/4934828 0 重标度时间Ito积分[闭合] KateAsks公司 https://math.stackexchange.com/users/1056303 2024-06-19T14:55:02Z年 2024-06-23T13:01:38Z <p>我想知道关于差速器可以说些什么</p><p>$$dB_{\phi(t)}$$</span>对于某些连续可微函数$\phi:[0,\infty)\to[0,\ infty()$</span>,其中$\phi'(t)&gt;0$</sspan>。含义:是否有一个特殊版本的Ito公式用于这些重新校准</p><p>我想也许$dB_{\phi(t)}=\sqrt{\phi'</p>(第页)<p>谢谢你</p>(第页)<p>编辑:让我把库尔特·G建议的想法计算出来</p>(第页)<p>设$W$定义为$$dW_t=\frac{1}{\sqrt{\phi´。后者意味着$W$</span>是鞅。此外:</p><p>$$\langle W\rangle=\langle\int_0^{\cdot}\frac{1}{\sqrt{\phi'(s)}}dB_{\phi(s){\rangle=\int_0 ^{\cf0}\frac{1}{\phi'(s)=id$$</span>因此,$W$</span>是一个布朗运动</p>(第页)<p>我们得出结论:$dB_{\phi(t)}=\sqrt{\phi'(t){dW_t$</span>,其中$W$</sspan>是布朗运动</p>(第页) https://math.stackexchange.com/q/4931606 1 会聚偏微分方程作为会聚粒子过程的流体力学极限 900边缘 https://math.stackexchange.com/users/884634 2024-06-12T19:26:08Z 2024-06-21T14:35:29Z <p>假设某个相互作用的粒子过程具有流体动力学极限,即PDE<span class=“math container”>$$\rho_t=L\rho+f\quad\quad\quad(1)$$</span>其中,$L$和$f$可能是非线性的。通过相互作用粒子过程,我的意思是每个$X^I_t$</span>都遵循某种SDE,其中漂移和扩散项可能是其他粒子(或其平均值)的函数。通过流体力学极限,我的意思是经验分布$\sum_{I=1}^n\delta_{X^I_t}$</span>在概率上收敛到解</p>(第页)<p>假设还有一系列PDE,其解收敛到满足PDE的解<span class=“math-container”>$$\rho^n_t=L^n\rho^n\quad\quad(2)$$</span>这样,在某种适当的意义上,<span class=“math-container”>$\rho^n(x,t)\rightarrow\rho(x,t)$</span>。假设每个<span class=“math-container”>$n$</span>都有一个流体动力学极限为$(2)$</sspan>的粒子过程。(请原谅在$X_t$</span>;$i$</sspan>和$n$</spa>上重复使用上标表示不同的意思。)</p><p>我们能否确定控制$X^n_t$</span>的SDE是否以某种方式收敛到控制$X_t$</span的SDE</p>(第页)<p>相关问题:如果我们知道某些过程收敛到极限过程,那么我们知道流体动力极限收敛吗</p>(第页) https://math.stackexchange.com/q/4935706网址 2 $\mathbb{R}^{2d}中Ito公式的$C^2$条件$ 罗德里戈M https://math.stackexchange.com/users/1071719 2024-06-21T12:13:16Z 2024-06-21T13:52:15Z <p>设$X,V\in\mathbb{R}^{d}$</span>,$Z=(X,V)$</sspan>,<span class=“math-container”>$g乘以d}$和:</p><p>$$dZ(t)=\begin{pmatrix}V\\g(t,X,V)\end{pmatrix}日期+\开始{pmatrix}0&amp;0\\\0(&amp;);\西格玛(t,Z){pmatrix}dW(t) $$</span>其中,$W$</span>是$2d$</sspan>维布朗运动</p>(第页)<p>Ito的公式为我们提供了一种方法,可以将$\phi(t,Z)$</span>写成一个Ito进程,前提是$\phi$足够正则。我对跟踪术语感兴趣,即$$tr\left[\begin{pmatrix}0&amp;0\\\0(&amp;);\西格玛(t,Z)\结束{pmatrix}\开始{pmatrix}0&amp;0\\\0(&amp;);\σ^T(T,Z)\end{pmatrix}\nabla^2\phi(T,Z)\right]$$</span>由于术语<span class=“math container”>$\nabla_{xx}^2 \phi$</span>乘以<span class=“math container”>$0$</span>,并且<span class=“math container”>$\nabla_{xv}^2 \phi$</span>被跟踪忽略,我们认为这等于<span class=“math container”>$$tr[\sigma(t,Z)\sigma^t(t,Z)\nabla_{vv}^2 \phi(t,Z)]$$</span></p><p>我的问题是,在这种情况下,我们真的需要<span class=“math-container”>$C^2(\mathbb{R}^{2d})$</span>对$\phi$的假设吗?直觉告诉我,我可能只需要C^1(\mathbb{R}^d)\乘以C^2(\mathbb{R{^d)$</span>中的$\phi\</p>(第页)<p>当然,如果第一个变量中没有$C^2$</span>,我可能无权将任何东西乘以$0$</sspan></p>(第页) https://math.stackexchange.com/q/4935655 0 作为概率模型的非齐次泊松过程如何具有1的跳跃大小? 斯特林 https://math.stackexchange.com/users/471973 2024-06-21T09:18:56Z 2024-06-21T13:14:12Z年 <p>我有一个事件发生的概率,发生在时间<span class=“math-container”>$t$</span>,$\operatorname{v}\left(t\right)$</sspan>,由以下等式建模:<span class=“数学容器”>$$\陶\,\压裂{{\rmd}\operatorname{v}\left(t\right)}{{\rmad}t}=-\operatorname{v}\left(t\right)+\tau a\sum{k}\delta\左(t-t{k}\右)$$</span>其中,<span class=“math-container”>$t{k}$</span>是从同质Poisson过程中提取的随机点,<span class=“math-container”>$a$</sspan>是跳跃大小,而$\tau$是衰减</p>(第页)<p>我的理解如下:</p><ul><li>如果泊松过程$t_{k}$不生成点,则$\operatorname{v}\left(t\right)$</span>线性衰减$\tau$</li><li>但如果泊松过程在$t$</span>处生成一个点,则$\operatorname{v}\left(t\right)$</sspan>会得到一个“跳转”</li><li>然而,我正在阅读的论文并没有将“跳转”限制在<span class=“math-container”>$1$</span>以下,并将跳转设置为$a=1$</sspan></li><li>但由于$\operatorname{v}\left(t\right)$</span>是一种概率,因此添加$+1$</sspan>需要<span class=“math-container”>$\operatorname{v}\left(t\right)&gt;1$</span>,这显然不可能,因为这是一种概率</li></ul><p>我这里错过了什么</p>(第页) https://math.stackexchange.com/q/4935598 0 将计数过程$N(t)$写成$N(t)=\sum_{k=1}^tY_k$:$Y_k美元的存在性和属性? 甲状旁腺素 https://math.stackexchange.com/users/190548网址 2024-06-21T06:01:52Z 2024-06-21T08:34:23Z年 <p>设$N(t)$<span class=“math-container”>$\{X_k\}$</span>为间隔时间产生的计数更新过程,间隔时间是独立的、同分布的正随机变量,平均值为>$\text{Var}(X_k)=\sigma^2$</span></p>(第页)<ol><li>对于一个非负整数$t$</span>,$N(t)$</sspan>可以写成$N(t)=\sum_{k=1}^tY_k$</sPA>,其中$\{Y_k\}$是独立的、同分布的、非负的、积分值的随机变量吗</li><li>如果是,什么是$\mathbb{E}(Y_k)$</span>和$\text{Var}(Y_k)$</li><li>如果$\mathbb{E}(X_k^3)&lt;\infty$然后$\mathbb{E}(Y_k^3)&lt;\infty$?(如果没有,什么条件就足够了?)</li></ol><p>动机:我试图对随机变量的随机数之和应用Berry-Esseen定理,使用$N(t)$</span>作为求和数。参见<a href=“的定理10.6https://link.springer.com/book/10.1007/978-3-642-15007-4“rel=”nofollow noreferrer“>Chen,Goldstein&Shao(2011)</a></p><p><strong>我尝试过的</p><p><em>对于(1)。设<span class=“math-container”>$Y_k$</span>是间隔$$(k-1,k)$</sspan>期间到达的数量。然后(1)满足</p>(第页)<p><em>对于(2)。设$\mathcal{N}(0,1)$</span>表示均值为0,方差为1的正态分布。写入$\nu=\mathbb{E}(Y_k)$</span>、$\tau^2=\text{Var}。应用<a href=“https://math.stackexchange.com/tags/central-limit-theorem/info“>经典(Lindeberg–Lévy)中心极限定理</a>到<span class=“math-container”>$N(t)$</span>,其中<span class=“math-container”>$$\裂缝{N(t)-\nut}{\tau\sqrt{t}}\\文本{分布收敛到}\\数学{N}(0,1)\标记{1}$$</span>作为<span class=“math-container”>$t\to\infty$</span>。然后将中心极限定理应用于计数过程(例如<a href=“https://ocw.mit.edu/courses/6-262-discrete-stochastic-processes-spring-2011/931ffa0940899c27f34b71ad64fd2bb0_MIT6_262S11_chap04.pdf“rel=”nofollow noreferrer“>Gallager,2011,定理4.3.2),其中<span class=“数学容器”>$$\裂缝{N(t)-t/\mu}{\sigma\mu^{-3/2}\sqrt{t}}\\文本{分布收敛到}\\数学{N}(0,1)\标记{2}$$</span>作为<span class=“math-container”>$t\to\infty$</span>。通过分布收敛极限的唯一性<em>和有力的手工节约</em>,推导出<span class=“math-container”>$$\nu=\frac{1}{\mu}\qquad(平方米)\τ=\frac{\sigma}{\mu^{3/2}}美元</span></p><p>作为交叉检验,布莱克威尔定理(例如<a href=“https://ocw.mit.edu/courses/6-262-discrete-stochastic-processes-spring-2011/931ffa0940899c27f34b71ad64fd2bb0_MIT6_262S11_chap04.pdf“rel=”nofollow noreferrer“>Gallager,2011,定理4.6.2)提供了<span class=“数学容器”>$$\lim{k\to\infty}\mathbb{E}(Y_k)=\压裂{1}{\mu}美元</span></p><p><em>对于(3)。我的直觉是考虑倒数随机变量<span class=“math-container”>$1/X_k$</span>,也许给它设一个下限</p>(第页)<p>参考文献</p><p>陈路易·H.Y。;拉里·戈尔茨坦(Larry Goldstein);邵琦曼(2011)<a href=“https://link.springer.com/book/10.1007/978-3642-15007-4“rel=”nofollow noreferrer“>Stein方法的正规逼近</p><p>Robert Gallager(2011)<a href=“https://ocw.mit.edu/courses/6-262-discrete-stochastic-processes-spring-2011/pages/course-notes/“rel=”nofollow noreferrer“>离散随机过程https://ocw.mit.edu/courses/6-262-discrete-stochastic-processes-spring-2011/931ffa0940899c27f34b71ad64fd2bb0_MIT6_262S11_chap04.pdf“rel=”nofollow noreferrer“>第4章:续签流程</p> https://math.stackexchange.com/q/4935321 0 Riemann-Liouville工艺和术语前的常数 gb2718标准 https://math.stackexchange.com/users/1311155 2024年6月20日下午4:24分55秒 2024-06-21T07:11:20Z <p>考虑以下形式的Riemann-Liouville过程:</p><p>$$B^H_t=c_H\int_0^t(t-s)^{-H-1/2}dB_s$$</p><p>为什么常数$c_H$</span>可以在不影响过程属性的情况下更改(因为它会更改其方差)</p>(第页)<p>到目前为止,我遇到了Riemann-Liouville过程的不同定义,其中一些定义为常数<span class=“数学容器”>$c_H=\frac{1}{\Gamma(1/2+H)}$</span>(参见Mandelbrot和Van Ness(1968)),另一些定义为<span class=“数学容器”>$c_H=\sqrt{2H}$</span>(参见Marinucci(1999)),甚至<span class=“数学容器”>$c_H=1$</span></p>(第页)<p>它们都是Riemann-Liouville过程的等效定义吗?或者它们应该被称为简单Volterra Gaussian过程的等效定义吗</p>(第页) https://math.stackexchange.com/q/4935594 0 关于随机变量向量 赫里托勒恩 https://math.stackexchange.com/users/1135572 2024-06-21T05:36:05Z 2024-06-21T05:36:05Z <p>在我的研究工作中,我遇到了以下表达式</p><p><span class=“math container”>$p=\bunderrace{A_1(\textbf{h} _2^T(n)\cdot\Psi\cdot\textbf{h} _1个(n) +A_2h_0(n))}_{第1部分}$</p><p>其中<span class=“math-container”>$\textbf{h} _2(n) $</span>是一个$M\乘以1$</sspan>向量,$\Psi$是一个标识矩阵和{h} _1个(n) $</span>是一个$M\乘以1$</sspan>向量</p>(第页)<p>标量$h2(n)$</span>表示为{h} _2+\Phi_{h2}(n)+h_{2_\epsilon}$</span>,标量<span class=“math-container”>$h_1(n{h} _1个+\Phi_{h_1}(n)+h_{1_\epsilon}$和{h} _0(0)+\Phi_{h_0}(n)+h_{0_\epsilon}$</p><p>此外</p><p>$A_1、A_2$是常量</p>(第页)<p>$w(n)$</span>是加性高斯白噪声,建模为$\mathcal{C}{n}(0,n_w)$</span</p><p><span class=“math-container”>$\hat{h}(小时)_{2} $</span>和<span class=“数学容器”>$\hat{h}(小时)_{1} $</span>是具有零均值和单位方差的复高斯随机变量</p>(第页)<p>$\Phi_{h_{2}(n)$</span>被建模为零均值圆对称复高斯(ZMCSCG),即$\mathcal{C}\mathcal{n}(0,(1-\rho_{h_2}}^{2(n-1)})\sigma^2_{h_2{}})$</span</p><p><span class=“math-container”>$h_{{2}_{\epsilon}}$</span>也被建模为ZMCSG,即<span class=“数学容器”>$\mathcal{C}\mathcal{N}(0,\sigma^2{\epsilon{h{2}}})$</span></p><p>$\Phi_{h_{1}}(n)$</span>建模为ZMCSCG,即$\mathcal{C}\mathcal{n}(0,(1-\rho_{h_1}}^{2(n-1)})</p><p><span class=“math-container”>$h_{{1}_{\epsilon}$</span>也被建模为ZMCSCG,即$\mathcal{C}\mathcal{N}(0,\sigma^2_{\epsilon_{h{1}})$</sspan></p><p>我的问题是,从(1)开始,我想以第1部分和$w(n)$</span>的方差比的形式编写表达式,但我不知道如何继续</p><p>在此方面的任何帮助都将不胜感激</p>(第页) https://math.stackexchange.com/q/4935583 -2 关于涉及随机变量的总和[闭合] 赫里托勒恩 https://math.stackexchange.com/users/1135572 2024-06-21T04:48:51兹 2024-06-21T04:48:51Z <p>在我的研究工作中,我得到了以下总结:</p><p>$p=\sum_{m=1}^m\{|h_{2,m}+\Phi_{h_2,m}}(n)+h_{{2,m}_{\epsilon}}|\cdot|h_{1,m}+\Phi_{h_1,m}}(n)+h_{{1,m}_{\epsilon}}|+H(h0+\Phi_{h0}(n)+H_{0_{\epsilon}})\}^2$</span>---(1)</p><p>其中</p><p>$H$是常量</p>(第页)<p>$h{2,m}$和$h{1,m}$</span>是具有零均值和单位方差的复高斯随机变量</p>(第页)<p>$\Phi_{h_{2,m}}(n)$</span>被建模为零均值圆对称复高斯(ZMCSCG),即$\mathcal{C}\mathcal{n}(0,(1-\rho_{h_2,m}}^{2(n-1)})\sigma^2_{h_2,1,m})$</span</p><p><span class=“math-container”>$h_{{2,m}_{\epsilon}$</span>也被建模为ZMCSCG,即,$\mathcal{C}\mathcal{N}(0,\sigma^2_{\epsilon_{h{2,m}})$</sspan></p><p>$\Phi_{h_{1,m}}(n)$</span>被建模为ZMCSCG,即$\mathcal{C}\mathcal{n}(0,(1-\rho_{h{1,m}}^{2(n-1)})</p><p><span class=“math-container”>$h_{{1,m}_{\epsilon}$</span>也被建模为ZMCSCG,即,$\mathcal{C}\mathcal{N}(0,\sigma^2_{\epsilon_{h{1,m}})$</sspan></p><p>我的问题是如何解方程(1),以便我能够替换$h{2,m}$</span>,$h{1,m}$</span>$h_container_{{2,m}_{\epsilon}$</span>和<span class=“math-container”>$h_{{1,m}_{\epsilon}}$</span></p>(第页)<p>在此方面的任何帮助都将不胜感激</p>(第页) https://math.stackexchange.com/q/4934770 0 不同概率空间中随机变量的条件期望 好奇心 https://math.stackexchange.com/users/1281006 2024-06-19T12:39:50Z 2024-06-21T04:38:48Z年 <p>设$(X,\mathcal{F} X(_X),\mathbb{P})$</span>和<span class=“math-container”>$(Y,\mathcal{F} 是(_Y),\mathbb{Q})$</span>是两个概率空间。我知道随机变量$Z:X\rightarrow\mathbb{R}$</span>的期望值受随机变量$W:Y\right箭头\mathbb{R}$的影响。所以我想使用条件期望-类似于<span class=“math-container”>$\mathbb{E}(Z\mid-W)$</span></p>(第页)<p>所以我的问题是:</p><ol><li><p>如果随机变量来自不同的概率空间,可以定义条件期望吗</p>(第页)</li><li><p>如果是这样,我会有条件期望的所有通常性质吗</p>(第页)</li><li><p>如果没有,我该怎么处理?我应该考虑两个概率空间的乘积吗</p>(第页)</li></ol> https://math.stackexchange.com/q/4935497 0 正随机过程到a.s.的充分条件的参考达到$+\infty$[已关闭] https://math.stackexchange.com/users/520710 2024-06-20T22:08:21Z年 2024-06-20T22:20:03Z年 <p>我想知道一个正随机过程满足p</p><p>任何人都可以在有这种结果的地方分享一些参考资料,或者在答案中列出结果吗</p>(第页) https://math.stackexchange.com/q/4934339 0 随机过程连续型的存在性 qp212223号 https://math.stackexchange.com/users/428839 2024-06-18T16:34:51Z 2024-06-20T13:37:45 Z <p>设$g(t,x)$</span>是开集上的连续函数</p>(第页)<p>设$X_t(X)$</span>是一个值随机域,其中$t\in[0,t]$</spa>和$X\in\mathbb{R}^d$</sspan>几乎可以确定是连续的</p>(第页)<p>假设<span class=“math container”>$$\textbf{(1)}\qquad\mathbb{E}\left[\int_0^T\mathbf{1}_{\partial S}(t,X_t(X))dt\right]=0$$</span>对于每个<span class=“math-container”>$X\in\mathbb{R}^d$</sspan><strong>我想知道这是否足以证明随机字段$$a_t(x)\equiv\int_0^t\mathbf存在连续版本/修改(在<span class=“math-container”>$(t,x)$</span>中{1} _秒(s,X_s(X))g(s,X_s(X))ds.$$</span></strong></p><p><span class=“math-container”>$\textbf{(1)}$</span>意味着$$\mathbb{p}\Big(A\text{在}(t,x)\text{对于[0,t]\Big中的每}t连续)=1$$对于每一个$</span>意味着,对于勒贝格来说,几乎可以肯定的是,在[0,T]$</sspan>,s\cup\text{int}(s^c)$</spa>中,几乎每一个$s(s,X_s(X),因为有无数这样的$x$</span>。我们当然可以得出结论,在勒贝格几乎每一个[0,t]\times\mathbb{R}^d$</span>中的$(t,x)都存在一个连续的版本。当然,这还不够</p>(第页)<p>任何帮助都将不胜感激</p>(第页)<p>如果有帮助,进一步假设$$dX_t(x)=b(t,x,x_t$$</span>$X_0(X)=h(X)$</span>。假设<span class=“math-container”>$x\in\mathbb{R}^k$</span>、$b$</sspan>、$\sigma$和$h$都是全局有界的,并且Lipschitz是连续的(因此存在映射的修改/版本\mapsto X_t(X)$</span>几乎可以肯定是连续的)。也可以假设<span class=“math container”>$$\sigma(t,x,y)\sigma(t,x,y)^t\succurlyeq\epsilonI$$</span>,其中<span class=“math container”>$\epsilon&gt;0美元</span></p>(第页) https://math.stackexchange.com/q/3631069 2 随机volterra过程的Ito公式 压碎 https://math.stackexchange.com/users/557565 2020-04-18T03:35:58Z 2024-06-20T12:11:23Z年 <p>我有点怀疑伊藤公式在以下随机过程中的应用(这是一个分数布朗运动过程)。假设<span class=“math-container”>$W_s$</span>是一个典型的布朗运动;\阿尔法&lt;1$</span>和<span class=“math-container”>$\kappa,\theta,\nu$</sspan>的参数。我们有这个过程<span class=“math-container”>\开始{方程式}V_t=V_0+\frac{1}{\Gamma(\alpha)}\int_0^t(t-s)^{\alpha-1}\kappa(\theta-V_s)ds+\frac{1}}{\Gamma(\ alpha\结束{方程式}</span>既然存在奇异积分,那么在这种情况下可以使用通常的Ito引理吗?我看到了在[0,t)$</span>中允许$s<span class=“math-container”>$g(s)=\frac{1}{\Gamma(\alpha)}(t-s)^{\alpha-1}$</sspan>的证明,上面的等式变成<span class=“math-container”>\开始{方程式}V_t=V_0+\int_0^t g(s)\kappa(\ta-V_s)ds+\int_0^t g\结束{方程式}</span>然后,证明的作者将Ito的公式应用于$\sigma_t=\sqrt{V_t}$</span>,它给出了<span class=“math container”>\开始{方程式}\σ_t=\sigma_0+\int_0^t(\frac{\kappa\theta}{2}\cdot g(s)-\frac}\nu^2}{8})\结束{方程式}</span>我确实知道如何应用通常的Ito引理,但问题是,这甚至是在奇异随机过程/volterra过程中应用Ito引言的正确方法吗?或者还有其他方法来构造伊藤公式吗。该公式已被用作金融中的一个应用程序,称为粗糙赫斯顿模型</p>(第页)<p>非常感谢您的指点/指导</p>(第页) https://math.stackexchange.com/q/4934829 0 超市家族的连续性 qp212223号 https://math.stackexchange.com/users/428839 2024-06-19T14:58:45 Z 2024-06-20T00:19:38Z <p>假设<span class=“math-container”>$\{Z_t(\alpha)\}_{\alpha}$</span>是类(D)的一个超鞅族,它是已知的联合连续的<em><strong></em>,几乎可以肯定地说,它是$(t,\alpha)$</sspan>的函数。在[0,t]\times\mathbb{R}^d$</span>中说$(t,\alpha)。Doob-Meyer分解表明,对于每个$\alpha$,都存在一个递增的可预测过程$A_t(\alpha)$</span>和一个鞅,这两个过程在<span class=“math-container”>$t$</sspan>中是连续的,使得<span class=“math container”>$$Z_t(\alpha)=M_t(\alpha)-A-t(\alpha)$$</span></p><p>几乎可以肯定(当然,空集可能依赖于<span class=“math-container”>$\alpha$</span>)。有人知道关于<span class=“math-container”>$M$</span>和<span class=”math-contacter“>$A$</sspan>在<span class=</p>(第页) https://math.stackexchange.com/q/4935009 0 伊藤扩散的随机场能在勒贝格测度0集合中花费正时间吗? qp212223号 https://math.stackexchange.com/users/428839 2024-06-19T21:10:46Z年 2024-06-19T21:10:46Z年 <p>假设我有一个由以下SDE管理的Itódiffusion家族:$$dX_t(x)=b(t,x,x_t$$</span>$X_0(X)=h(X)$</span>。假设<span class=“math-container”>$x\in\mathbb{R}^d$</span>,$x_t(x)$</sspan>值为$\mathbb{R}^k$>$h$</span>都是全局有界的且Lipschitz连续的(因此存在映射$(t,x)\mapsto x_t(x)$</sspan>的修改/版本,它几乎是连续的)。进一步假设<span class=“math-container”>$$\sigma(t,x,y)\sigma;0美元</span></p>(第页)<p>设<span class=“math-container”>$S\subseteq[0,\infty)\times\mathbb{R}^k$</span>是具有Lebesgue测度的闭集。波动矩阵的条件意味着$X_t(X)$$\textbf{(1)}\qquad\int_0^\infty\mathbf具有密度,因此对于每个{1} _秒(t,X_t(X))dt=0$$</span>几乎可以确定每个<em><strong>固定<span class=“math-container”>$X\in\mathbb{R}^d$</sspan></strong></em>(即,不发生这种情况的空集取决于<span class=“math-container”>$X$</spa>)</p>(第页)<p>我假设的关于$X_t(X)$</span>的条件是否足以说明对于每个<span class=“math-container”>$X\in\mathbb{R}^d$</sspan>都成立?即,是否存在一个可测量的集合$a$</span><span class=“math container”>$$\mathbb{P}\left(A\right)=1\\A\subseteq\left\{\omega:\int_0^\infty\mathbf{1} _秒(t,X_t(X,ω))dt=0\四\所有\;x\in\mathbb{R}^d\right\}\quad$$</span></p> https://math.stackexchange.com/q/4933826网址 5 等价鞅测度中的最优运输距离 杰弗里风 https://math.stackexchange.com/users/41972 2024-06-17T15:40:33Z 2024-06-19T19:43:13Z年 <p>我对这个想法已经研究了一段时间,可以使用一些指导。现在我想知道我写的公式是否有意义。非常感谢您的帮助</p>(第页)<p><strong>问题陈述想法</p><p>设$X_t$和$Y_t$是由$$dX_t=\mu_X(t,X_t)dt+\sigma_X(t,X_t)dW_t^{\mathbb{P}}$$</span>$$dY_t=\mu_Y(t,Y_t)dt+\sigma_Y(t,Y_t)dW_t^{\mathbb{P}}$$</span>其中$dW_t^{\mathbb{P}$</span>是基测度下的布朗运动。我遗漏了很多背景定义。为此,我们应该知道Itóprocess是马尔可夫过程,并且假设漂移项和扩散项的某些正则性条件,可以通过它们的转移密度函数唯一地识别</p>(第页)<p>Girsanov理论</p><p>根据Grisanov理论,存在两个等价的Martigale测度{Q} X(_X)$</span>和$\mathbb{Q} 是(_Y)$</span>将流程转换为等效的martignale度量(风险中性度量)</p><p>$$dX_t=\sigma_X(t,X_t)dW_t^{\mathbb{Q} X(_X)}$$</span>$$dY_t=\sigma_Y(t,Y_t)dW_t^{\mathbb{Q} 是(_Y)},$$</span></p><p>有效地消除过程中的漂移。对于基本度量中的过程,$X_t$和$Y_t$的跃迁密度我们写为$p_t^X(X)$和$p_t Y(Y)$对于<span class=“math-container”>$\mathbb下过程的转移密度{Q} X(_X)$</span>和$\mathbb{Q} 是(_Y)分别为$</span></p>(第页)<p>Radon-Nikodym衍生物</p><p>度量的变化是通过Radon-Nikodym导数<span class=“math container”>$Z_t^X(X)=\frac{d\mathbb{Q} X(_X)}{d\mathbb{P}}$</span>和$Z_t^Y(Y)=\frac{d\mathbb{Q} 是(_Y)}{d\mathbb{P}}$</span>,由定义</p><p><span class=“math container”>$$Z_t^X(X)=\exp\left(−\int_0^t\frac{\mu_X(X_s,s)}{\sigma_X(X_s,s)}dW_s^{\mathbb{p}}−\frac{1}{2}\int_0^t\left(\frac{\mu_X(X_s,s)}{\sigma_X(X_s,s)}\right)^2 ds\right)$$</span><span class=“math-container”>$$Z_t^X(y)=\exp\left(−\int_0^t\frac{\mu_y(y_s,s)}{\sigma_y(y-s,s){dW_s^{\mathbb{P}}−\frac{1}{2}\int_0 ^t\left$$</span></p><p>在这之前,我非常确信这些定义是根据Girsanov理论表述的,并且是正确的。如果错了,请纠正我。下一步我有点不确定</p>(第页)<p><strong>过渡密度测量值的变化</p><p>Radon-Nikodym导数可以这样写$$Z_t(x)=\frac{d\mathbb{Q}}{d\mathbb{P}}=\frac{Q_t$$</span>其中,$q_t(x)$</span>和$p_t(x)$</span>是与度量值相关的概率密度函数。然后,我可以用基本测度和Radon-Nikodym导数写出鞅测度的转移密度</p><p>$$q_t^X(X)=p_t^X$$</span><span class=“数学容器”>$$q_t^Y(Y)=p_t^Y(Y)Z_t^Y(Y)$$</span></p><区块报价><p>在此之前,我不应该说任何不同寻常的话,所以我只想知道在这之前我的数学是否可以接受要点</p>(第页)</blockquote><p>这就是我想说的</p><p>Wasserstein距离</p><p>在度量空间$(M,d)$</span>上,将两个概率测度之间的Wasserstein-1距离定义为</p><p>$$W_1(\mu,\nu)=\inf_{\gamma\in\gamma(\mu,\nu)}\int_{M\times M}d(x,y)\gamma(x,y)dxdy$$</p><p>其中<span class=“math-container”>$\Gamma$</span>是$\mu$和$\nu$的所有<em>耦合</em>的集合。耦合是一种联合概率分布,具有$\mu$和$\nu$作为边际。这被解释为找到最小的运输成本</p>(第页)<p><strong>这就是我想说的</p><p>让我们在基本度量值$\mathbb{p}$、$p_t^X(X)$和$p_t Y(Y)$中,写出$X_t$和$Y_t$的跃迁密度之间的Wasserstein-1距离</p>(第页)<p>$$W_1^{mathbb{p}}(X_t,Y_t</p><p>我在这里强调,$\gamma_{\mathbb{p}$</span>位于带有下标的度量值中。<span class=“math-container”>$\gamma$</span>耦合也被称为传输映射,因为它们被解释为将质量从一个概率密度指向另一个概率浓度的图</p>(第页)<p>因此,这是一大步<strong>我想知道,通过将Radon-Nikodym导数应用于<span class=“math-container”>$\gamma_{\mathbb{Q}$</span>,可以在鞅测度下实现耦合>$\gamma_{\mathbb{P}}$</span></strong>如果是这样,我可以写下:</p><p>$$\gamma_{\mathbb{p}}(x,y)Z_t^x$$</span>其中$\gamma_{\mathbb{Q}}\in\gamma\left(Q_t^X(X),Q_t^Y(Y)\right)$</span>。如果这一步骤是允许的,那么我就可以在<span class=“math-container”>$\mathbb中写入运输成本{Q} X(_X)\次\mathbb{Q} 是(_y)$</span>作为基本度量中运输成本的函数$$\int_{\mathbb{R}^n\times\mathbb}R}^n}d(x,y)\gamma_{\mathbb{Q}}></p><p><strong>验证尝试</strong></p><p>据我所知,概率密度函数与测度理论的测度非常相似,似乎可以互换或完全相同。因此,对于下面的证明,我将停留在概率密度领域。概率密度和测度之间的联系是我仍在学习更多的东西,我喜欢这里的任何方向。有趣的是,据我所知,根据上面给出的关系,氡-尼可金导数(测量值的变化)也适用于密度和测量值。在下文中,使用联合概率密度代替测量理论中的耦合,我认为这是正确的替代。然后,边际密度与测度理论的边际测度类似</p>(第页)<p>设$p(x,y)$</span>是具有边距的联合概率密度,即$$f_X(X)=\n int p(X,y)dy$$$</span>和$$f_Y(Y)=\int p(x,Y)dx$$$</span></p>(第页)<p>设$L(x)=q_x(x)/f_x(x)$</span>是作为测度变化的Radon-Nikodym导数,将密度$f_x$发送到$q_x$。设<span class=“math container”>$R(y)=q_y(y)/f_y(y)$</span>是Radon-Nikodym导数,它将密度<span class=“math container”>$f_y$</span>发送到<span class=“math container”>$q_y$</span></p>(第页)<p>设$$g(x,y)=p(x,y)L(x)R(y)$$</p><p>声明是$g(x,y)$</span>是一个有效的联合概率密度,$g(x,y)$的边距是$q_x(x)$</span</strong></p><p><strong>1.</strong>$g(x,y)$</span>是有效的接头密度,因为<span class=“math-container”>$$\int\int g(x、y)dx dy=1$$</sspan>。证明(使用总概率法):$$\int\int p(x,y)L(x)R(y)dx dy=\int R(y$$</span>$$=\int R(y)f_y(y)\left(\int p(x|y)L(x)dx\right)dy$$</p><p>看看括号内的积分,我们首先注意到,根据联合密度的定义,$\int p(x|y)dx=1$</span>,因此$p(x| y)$</sspan>可以作为概率密度。Radon Nikodym导数将一个有效密度转换为另一个,因此该积分积分为1,<span class=“math-container”>$\int p(x|y)L(x)dx=1$</span>。$$=\t R(y)f_y(y)\left(\t p(x|y)L(x)dx\right)dy=\t R$$</span>然后对这个积分提出了同样的论点:有效概率密度积分为1</p><p><strong>2计算$g(x,y)$</span>的边距。$$\int g(x,y)dx=\int p(x,y)L(x)R(y)dx=R(y$$</span>通过与上一步相同的参数,$\int p(x|y)L(x)dx=1$,那么$$\int g(x,y)dx=R(y)f_y(y)=\frac{q_y(y)}{f_y$$</span>一个相同的论点表明$$\int g(x,y)dy=q_x(x)$$</p><p>量化宽松政策</p><p>这些都允许吗?我最应该注意哪些方面?非常感谢您的反馈</p>(第页)<p>谢谢</p>(第页) https://math.stackexchange.com/q/4934888 -1 二阶段递归随机过程 安东尼奥·里格纳内斯 https://math.stackexchange.com/users/1339263 2024-06-19T16:50:08Z 2024-06-19T16:50:08Z <p>我正在学习如何使用Markowitz MVO优化投资组合,并使用递归的两阶段随机过程。我所做的是:</p><ol><li>获取资产价格</li><li>把它们调到原木上</li><li>得到了日志返回</li><li>使用arima来检查它们是什么过程(都是arima(000))</li><li>由于所有分布均为正态分布,我将其平均值和Sd用于蒙特卡罗,为每个资产生成1000个模拟</li><li>得到了每个仿真距离的25%、50%和75%的值,以创建三个场景</li><li>我用AMPL制作了模型,你可以在图片中看到</li></ol><p>由于这个问题变得不可行,我想知道我是否犯了任何错误(作为一名新手)</p>(第页)<p>谢谢你</p>(第页)<p>附言。最低回报率如图中所示,但我<a href=“https://i.sstatic.net/pn4t6vfg.png“rel=”nofollow noreferrer“>在此处输入图像描述</a>在我运行模型时正确包含了它</p> https://math.stackexchange.com/q/4934766 1 步长增加的有偏随机游动的期望停止时间 洛伦佐 https://math.stackexchange.com/users/1328988 2024-06-19时间12:34:55 z 2024-06-19T12:34:55 Z <p>设$S_n$</span>是一个随机过程,其中<span class=“math-container”>$$S_{n+1}=S_n-1+\开始{cases}n^2&amp;\mathrm{with\;probability\;}\frac{1}{2}\\-n^2(&amp;2);\数学{else}\结束{cases}。$$</span>美元</p><p>现在让$T:=\inf\{n\geq0:S_n\leq0\}$</span>作为进程的停止时间,当它第一次变为负值时</p>(第页)<p>$\mathbb{E}(T)$</span>有限吗</p>(第页)<p>我知道,对于有点相关的随机过程(从<span class=“math-container”>$n\geq1$</span>开始)$$S'{n+1}=S'_n-\frac{1}{n^2}+\begin{cases}1和amp;\mathrm{with\;probability\;}\frac{1}{2}\\-1和amp;\数学{else}\结束{cases},$$</span>以及类似的停止时间<span class=“math container”>$T'$</span>,它认为<span class=“math container”>$\mathbb{E}(T')=\infty$</span>(因为<span class=“math container”>$\sum^\infty_n\frac{1}{n^2}=2$</span>)</p>(第页)<p>而&quot;之间的比率;常数部分&quot;和&quot;二次部分&quot;对于这两个进程来说都是一样的,直观地看,对于进程$S_n$</span>,负方向所需的步骤数减少了,而对于$S'{n+1}$</sspan>,则保持不变</p>(第页)<p>此外,矛盾证明,如<a href=“https://math.stackexchange.com/questions/1085399/expectation-of-hitting-time-for-simple-symmetric-random-walk#comment2216350_1085399“>这个问题不起作用</p><p>如何显示$\mathbb{E}(T)&lt;\infty$</p>(第页) https://math.stackexchange.com/q/4934184 0 为什么Mathematica模拟随机过程的首次命中时间与理论结果不匹配? 普尔泽莫 https://math.stackexchange.com/users/99778 2024-06-18T11:07:26Z 2024-06-19T11:16:51Z <p>设$\theta,\sigma,\mu$</span>为正实数。我们考虑以下均值回复随机过程$d X_t=\theta\cdot(\mu-X_t)dt+\sigma d B_t$,其中$X_0=X$。这里是布朗运动。该过程被称为奥恩斯坦-乌伦贝克过程。它是均值回复,其性质是众所周知的。特别地,已知存在两个线性独立的特征函数<span class=“math container”>$F_r(x)$</span>和<span class=“math container”>$G_r(x)$</span>,对于特征值<span class=“math container”>$r&gt;0$</span>,无穷小生成器的${\mathfrak G}_x:=\theta(\mu-x)d/dx+\sigma^2/2\cdot d^2/d x^2$</sspan>。这些本征函数都是正的,第一个严格递增,另一个严格递减。这些特征函数表示:</p><p><span class=“math-container”>\开始{eqnarray}F_r(x)&amp=&amp;\int\limits_0^\infty u^{\zeta-1}\cdot e^{A(x-\theta)u-\frac{u^2}{2}}du&amp=&amp;2^{\frac{\zeta}{2}-1}\左(\伽马\左(\frac{\zeta}{2}\右)\,_1F_1\左(\frac{\zeta}{2};\frac{1}{2{;\frac{1}[2}A^2(x-\theta)^2\右)+\sqrt{2}A\Gamma\左(\frac{\zeta+1}{2}\右)(x-\theta)\,_1F_1\左(\压裂{\泽塔+1}{2};\压裂{3}{2{;\裂缝{1}{2neneneep A^2(x-\θ)^2(右)\\G_r(x)&amp=&amp;\int\limits_0^\infty u^{\zeta-1}\cdot e^{A(\theta-x)u-\frac{u^2}{2}}du&amp=&amp;2^{\frac{\zeta}{2}-1}\左(\伽马\左(\frac{\zeta}{2}\右)\,_1F_1\左(\frac{\zeta}{2};\frac{1}{2{;\frac{1}[2}A^2(x-\theta)^2\右)+\sqrt{2}A\Gamma\左(\frac{\zeta+1}{2}\右)(\theta-x)\,_1F_1\左(\压裂{\泽塔+1}{2};\压裂{3}{2{;\裂缝{1}{2neneneep A^2(x-\θ)^2(右)\结束{eqnarray}</p><p>其中<span class=“math-container”>$\zeta:=r/\mu$</span>和$A:=\sqrt{(2\mu)/\sigma^2}$</sspan></p>(第页)<小时/><p>现在,根据《K Ito》第4.6节,Henry p McKean Jr;扩散过程及其采样路径;,拉普拉斯变换$E_x\left[E^{-r\cdot\tau_b}\right]:=第一次击中时间$\tau_b:=inf(t&gt;0|x_t&gt;b)$</span>的E\left[E^{-r\cdot\tau_b}|x_0=x\right]$分布采用以下形式:</p><p><span class=“math-container”>\开始{eqnarray}E_x\left[E^{-r\cdot\tau_b}\right]=\左\{\开始{数组}{lll}\压裂{F_r(x)}{F_r(b)}&amp;\mbox{if$x\le b$}\\\裂缝{G_r(x)}{G_r(b)}&amp;\mbox{if$x&gt;b$}\\\结束{数组}\对。\标记{1}\结束{eqnarray}</p><p>我们选择使用Mathematica通过蒙特卡罗模拟验证结果$(1)$</span></p>(第页)<p>我们取$\mu,r,\sigma,\theta=5,3,3,1.1$</span>和第一个值$x=1/2$</sspan>。然后,我们在时间网格上模拟了有问题过程的<span class=“math container”>$num=100005000$</span>实例<span class=“math container”>$t=i\cdot-dt$</span>,其中<span class=“math container”>$i=0,\cdots,\lfloort_{max}/dt\lfloor$</span>,其中<span class=“math container”>$dt=0.01$</span>和<span class=“math container”>$t_{max}=10$</span>。这里使用的命令是<code>ItoProcess[]</code>以及<code>RandomFunction[]</code>。对于进程l1的每个实例,我们通过命令<code>l2=FoldList[Max,l1]</code>构造了它的运行最大值,然后通过找到方程的(唯一)根来确定屏障的第一次击中时间。然后,我们对这些随机时间内$(1)$</span>的左手边取平均值,最后将结果绘制为屏障的函数</p>(第页)<p>代码中的定义:</p><pre><code>(*这是我们严格增加的\无穷小运算符。*)F[x_,\[Mu]_,\[西格玛]_,\[θ]_,r_]:=[{\[Zeta]=r/\[Mu],A=Sqrt[(2\[Mu])/\[Sigma]^2]},2^(-1+\[泽塔]/2) (伽玛[\[Zeta]/2]超几何c1F1[\[Zeta]/2,1/2,1/2 A^2(x-\[Theta])^2]+Sqrt[2]A(x-\[Theta])伽马[(1+\[Zeta])/2] 超几何1F1[(1+\[泽塔])/2,3/2,1/2 A^2(x-\[Theta])^2])];(*在\Ornstein-Uhlenbeck工艺。*)清除[\[Mu],\[Sigma],\[θ],x,t,dt];x0=1/2;proc=ItoProcess[\[DifferentialD]x[t] ==\[Theta](\[Mu]-x[t])\[微分D]t+(\[西格玛]x[t] )\[微分D]w[t],x[t]、{x、x0}、t、,w\[分布式]WienerProcess[]];</code></pre><p>下面给出了正在使用的代码主体</p>(第页)<pre><code>{\[Mu],r,\[Sigma],\[Theta]}={5,3,Sqrt[9],1.1};数量=5000;dt=0.01;tmax=10;rf=随机函数[proc,{0.,tmax,dt},num,方法-&gt&quot;KloedenPlatenSchurz&quot;];mLL={};bs=数组[#&amp;,100,{1/2,5}];做[b=bs[[其中]];mll={};做[l1=rf[[2,1]][[p,All]];l2=文件夹列表[Max,l1-b];f=插值[l2];tind=吨/。查找根[f[t]==0,{t,1,Floor[tmax/dt]+1}];如果[!NumberQ[tind]||!(1&lt;=tind&lt;=地板[tmax/dt]+1),{tstar,mExp}={NaN,NaN},tstar=dt*(tind-1);mExp=经验[-r*tstar];];mll=连接[mll,{tstar,mExp}}];,{p,1,长度[rf[[2,1]]}];mll1=选择[mll,数字Q[#[[2]]]&amp;];mLL=连接[mLL,{{b,平均值[mll1[[All,2]],Sqrt@方差[mll1[[全部,2]],长度[mll1]/Length[mll]}];PrintTemporary[&quot;b=&#quot;,N@bs[[which]];已完成&【】;,{which,1,长度[bs]}];</code></pre><p>代码的绘图部分:</p><pre><code>ListPlot[{{#[[1]],Around[#[2]],#[[3]]}&amp;/@mLL,{#,F[x0,\[Mu],\[Sigma],\[θ],r]/F[#,\[Mu],\[Sigma],\[Cheta],r]}&amp;/@bs},轴标签-&gt;{“b”;,&quot;\\(\*SubscriptBox[\(E\),\(x\)]\)[实验[-r\!\(\*ubscriptBox[\(\\[头]\),\(b\)]]&quot;},绘图图例-&gt;{“MC”,“理论”}]</code></pre><p>蒙特卡罗模拟的结果(蓝色点和蓝色误差条)与理论曲线重叠,如下图所示</p>(第页)<p><a href=“https://i.sstatic.net/6rkeFNBM.png“rel=”nofollow noreferrer“><img src=”https://i.sstatic.net/6rkeFNBM.png“alt=”OU过程中第一次命中时间分布的拉普拉斯变换。这里,蓝色点和蓝色误差条代表模拟的平均值和标准偏差,而橙色点代表理论。“/></a></p><p>显然,模拟与理论不符!对于大值的<span class=“math-container”>$b$</span>,与理论结果相比,模拟的首次命中时间系统地太小。当我们增加实例数num时,结果似乎不会改变,也不会随着时间span<span class=“math-container”>$t_{max}$</span>的增加而改变</p>(第页)<p>这是什么原因</p>(第页) https://math.stackexchange.com/q/4908881 0 使用斯特林公式证明极限行为 篝火 https://math.stackexchange.com/users/1034702 2024-05-01T13:23:05Z 2024-06-19时间9:58:18Z <p>我正试图理解/证明一个引理,这是在两篇关于分支过程的论文中提出的(Jabbour-Hattab的《鞅与二叉搜索树的大偏差》中的引理2.2和Chauvin的《二叉树的鞅、嵌入和倾斜》中的引理2.8)。论文引用了斯特林公式和标准分析方法,但没有给出更多的证明</p>(第页)<p>对于<span class=“math-container”>$n\geq1$</span>和$C_0(z)=1$</sspan>,我们得到了一个乘积$C_n(z,当<span class=“math-container”>$n$</span>趋于无穷大时,我们有以下渐近行为:</p><p>$C_n(z)\sim\frac{n^{2z-1}}{\Gamma(2z)}$</p><p>不幸的是,当涉及到斯特林公式或伽玛函数时,我没有太多知识。任何帮助都将不胜感激</p>(第页) https://math.stackexchange.com/q/4934696 -1 离散时间随机过程的解析公式 赫尔姆08 https://math.stackexchange.com/users/1338952 2024-06-19T09:35:58Z 2024-06-19T09:35:58Z <p>我想问你一个数学问题。我想计算一个过程的动力学,这个过程在每次采样时都会有一个下降(比如等于40%),每次下降后都会有反弹(比如直到95%):这样就可以考虑到永久和暂时的影响。对于这条向下曲线在时间t的预期值,是否有一个解析的闭合解</p>(第页)<p>没有反弹的简单情况可以用一个乘法二项式过程来描述,其中$V_0$</span>作为过程的初始值,在每个采样步骤t,过程$V_t$</sspan>是<span class=“math-container”>$$V_t=V{t-1}*(1-D_t)$$</span>,其中$D_t$</sspan>等于所有t的常数0.4。因此,该过程可以描述为$$V_t=V_0*(0.6)^t$$</p><p>感谢所有试图帮助我的人</p> https://math.stackexchange.com/q/4934444 1 续签奖励流程的奖励尾概率 hshlmh公司 https://math.stackexchange.com/users/1338561 2024-06-18T20:15:21Z 2024-06-18T20:39:36Z <p>假设给我们一个骰子,它有$K$</span>个面,用<span class=“math-container”>$K=1,\dots,K$</span>表示,其中实现一个面的概率为[0,1]$$p_K\,其中[span class=“math-container>$sum_{K=1、\dots、K}p_K=1$</sspan>。现在,我们反复掷骰子。当实现一个面$k$</span>时,我们需要等待一个正随机时间$S_k$</span>,然后再滚动。问题是:给定总时间预算<span class=“math-container”>$B&gt;0$</span>,我们滚动了多少次特定的面<span class=“math-container”>$k^*$</p>(第页)<p>这个问题可以形式化为<a href=“https://en.wikipedia.org/wiki/Rewal_theory#续订-reward_processes“rel=”nofollow noreferrer“>更新-奖励过程</a>,其保持/等待时间是IID随机变量的序列$\{S_{k_i,i}\}_{i\geq1}$</span>,平均值为(0,\ infty)美元</span>。停止时间<span class=“math-container”>$\tau=\inf\{t=1,2,\dots\colon J_t=\sum_{i=1,\dots,t}S_{k_i,i}&gt;B\}=\sum_{t=1,2,\dots}1_{S_{t-1}\leq B}$</span>。然后,奖励被定义为实现face<span class=“math-container”>$k^*$</span>的次数。形式上,奖励是$\tau_{k^*}:=\sum_{t=1,2,\dots}1_{S_{t-1}\leq B}1_{k_t=k^*}$</span></p>(第页)<p>作者:<a href=“https://link.springer.com/book/10.1007/b97236“rel=”nofollow noreferrer“>Asmussen公式6.2(Wald方程)中的应用概率和排队,我们得到了<span class=“math-container”>$\mathbb{E}[\tau_{k^*}]=p_{k*}\mathbb{E}[\tau]$</span>,并且根据Prop.6.2,我们得到<span class=S_{x_1,1}]}+\frac{\mathbb{E}[S_{x1,1}^2]}{\mathbb{E}[S_{x_1,1}]^2}$</p>(第页)<p>我的问题是</strong>奖赏<span class=“math-container”>$\tau_{k^*}=\sum_{t=1,2,\dots}1_{S_{t-1}\leqB}1_}k_t=k^*$</span>是否存在指数衰减的尾浓度不等式,其形式为]]\leq\exp[-\dots]$?这里,假设每个人脸的$s_k$是正的次指数随机变量,这意味着$\forall\lambda\colon|\lambda |&lt;\压裂{1}{\alpha_k}\colon\mathbb{E}[\exp[\lambda S_k]\leq\exp[\frac{\lambda^2 v_k^2}{2}]$https://www.stat.cmu.edu/%7Earinaldo/Teaching/36709/S19/Scribed_Leachs/Feb5_Aleksandr.pdf“rel=”nofollow noreferrer“>(此处定义5.1)</a></p><p>我试图构造一个鞅$\sum_{t=1,2,\dots}1_{S_{t-1}\leqB}(1_{k_t=k^*}-p_{k^*{)$</span>并应用https://en.wikipedia.org/wiki/Azuma%27s_inquality(英文)“rel=”nofollow noreferrer“>Azuma-Hoeffing。然而,我不知道如何将Azuma-hoeffing中的固定离散时间$N$</span>与我们问题中的随机停止时间$\tau$</sspan>联系起来</p> https://math.stackexchange.com/q/2948704 0 计算E(Xt)、Var(Xt”)和Cov(ts,Xs) 汤12 https://math.stackexchange.com/users/596258 2018年10月9日T15:47:39Z 2024-06-18T19:02:55 Z <p><a href=“https://i.sstatic.net/su32v.png“rel=”nofollow noreferrer“><img src=”https://i.sstatic.net/su32v.png“alt=”在此处输入图像描述“></a></p><p>我是研究B.M.的新手,我很难理解下面的计算是如何得出的。我不确定Var,Cov,但我想推导E(Xt)的答案需要考虑西格玛和μ是常数这一事实。我了解B.M.的基本属性,但我很难理解如何正确应用它们</p>(第页) https://math.stackexchange.com/q/4934309 4 难以理解障碍选项 蒂莫菲耶8384 https://math.stackexchange.com/users/734943网址 2024-06-18T15:52:49 Z 2024-06-18T15:52:49 Z <区块报价><p>我们有以下包含屏障选项的合同:</p><ul><li>如果在合同期间,$S_t$</span>高于障碍水平$B$</sspan>,我们将收到$N\cdot\max(S_t-4.45,0),4.45&gt;B$</span>来自银行,其中<span class=“math-container”>$N$</span>-名义价值</li><li>如果在合同期内,$S_t$</span>低于$B$</sspan>的水平,那么如果<span class=“math-container”>$S_t\ge4.65$,我们将从银行收到$N\cdot(S_t-4.65)$,否则我们将向银行付款>$N\cdot(4.65-S_T)$</span></li></ul></blockquote><ol><li>推导合同参数必须满足的条件,使其在签订时的值为零,即平衡构成本合同的工具的当前值</li><li>创建一个脚本,该脚本将根据此合同的其余参数确定$B$</span>的值</li></ol><p><strong>我对问题的理解:</strong><strong>1.</strong>该合同包括一个直接购买障碍期权,如果$S_t$</span>低于$B$</sspan>,该期权将失效。然后激活期权,包括向银行发行的看跌期权和我们以4.65美元的相同行权价格购买的看涨期权。因此,在签订合同时,应满足以下条件:$$\text{call}K_1=\text{call}K_2-\text{put}K_2$$</p><p>然而,我不知道如何对障碍期权进行折现,我也没有在文献中找到任何帮助。有人能告诉我该用什么吗,或者给我一些小费吗</p>(第页)<p><strong>2.我们的合同是美国式的障碍选项。因此,最终付款取决于合同期内$\min{S_t}$和$B$之间的关系。对于这项任务,我已经给出了市场数据,我可以使用vanna-volga方法计算各种值,但我完全不知道如何确定$B$</span>屏障的值。我不希望任何人为我编写代码,只需要向我解释一下我可以使用什么条件来得出$B$</span>的公式</p>(第页) https://math.stackexchange.com/q/208111 0 概率与距离成反比 拉胡尔·纳拉帕 https://math.stackexchange.com/users/409006 2017年1月22日T00:45:40Z 2024-06-18T07:01:25Z <p><a href=“https://i.sstatic.net/WS7cC.png“rel=”nofollow noreferrer“>教科书问题</a></p><p>在我的一个家庭作业问题中,我很难理解这样一句话:“概率与从当前位置”</p><p>这三个距离分别是3厘米、4厘米和5厘米,我必须根据这个反比例规则为这三个事件分配概率,总计1</p>(第页)<p>我构造了以下方程式:</p><p>(1/3)x+(1/4)x+</p><p>然后,我求解x,得到x=(60/47),并将其乘以(1/3)、(1/4)和(1/5)。我的基本原理是找到一个常数因子,这样我就可以将概率分配给这三个事件,总计1,但我认为这是不对的</p>(第页) https://math.stackexchange.com/q/4933196 4 平稳随机过程子序列的收敛性 核武器 https://math.stackexchange.com/users/802456 2024-06-16T08:16:52Z 2024-06-17T18:08:07Z <p>设$\{X_n\}_{n=1}^\infty$</span>是一个实值平稳随机过程,而$\{W_n\}_{n=1}^\infty$是一个二值随机过程,其中。我们将<span class=“math container”>$W_n$</span>称为事件选择过程,因为<span class=“math container”>$W_n=1$</span>标记事件的发生</p>(第页)<p>将$K_i$</span>定义为第次$i$</span次$W_n=1$</span,即$K_i$</span是第次关键事件发生的时间:<span class=“math-container”>$$K_i=n\iff W_1+W_2+\cdots+W_n=i,W_n=1$$</span>成对<span class=“math-container”>$(\{K_i\},\{X_n\})$</span>现在是标记点过程的一个示例</p>(第页)<p>假设<span class=“math-container”>$Y_i=X_{K_i}$</span>,即,$Y_i$</sspan>是第$i$</span个事件发生时进程的值</p>(第页)<p><strong>问题陈述:证明序列$\{Y_i\}_{i=1}^\infty$</span>在分布中收敛,因为<span class=“math-container”>$\{X_n\}$</sspan>和$\{W_n\}$是联合平稳的</p>(第页)<p>最后,我在寻找一些条件(施加在<span class=“math-container”>$X_n$</span>、<span class=”math-continer“>$W_n$</span>和<span class=”math-container“>$K_I$</sspan>上),这些条件意味着序列$\{Y_I\}_{I=1}^\infty$是遍历的,即$$\lim_{N\rightarrow\infty}\frac{1}{N}\sum_{i=1}^Ng(Y_i)=g(Y_ \infty)$$</span>其中<span class=“math-container”>$Y_\infty$</span>是一个随机变量,$Y_i$</sspan>向其收敛,而$g$</spa>是一些足够好的函数。但首先需要$Y_i$</span>的收敛性和存在性</p>(第页)<p><strong>我尝试的解决方案:</strong>我展示了以下不太一般的结果</p><ul><li>如果<span class=“math-container”>$\{W_n\}$</span>和$\{X_n\}$是独立的,并且$X_n$</sspan>是固定的,那么$Y_i$</span>是相同分布的</li><li>如果<span class=“math-container”>$X_n$是i.i.d.,并且$W_n$通过$W_n=\mathbb依赖于它们{我}_{X_n\在B}$中,其中$\mathbb{I}$是指示符函数,而$B$是任意集(在潜在概率空间中可测量),然后$Y_I$</span>均匀分布</li></ul><p>显然,相同分布的$Y_i$</span>意味着它们(在分布中)收敛</p>(第页)<p>我处理更一般情况的方法包括查看概率$$P(Y_i\in A)=\sum_{n=1}^\infty P(X_n\in A,K_i=n)\\=sum_{n=1}^系数P(X_n在A中,W_1+cdots+W_{n-1}=i-1,W_n=1)$$</span>并应用一些时移(由<span class=“math container”>$X_n$</span>和<span class=“math container”>$W_n$</span>的联合平稳性保证)来形成某种递推关系。通过这个循环,我可以证明$P(Y_I\in A)$</span>收敛为$I\rightarrow\infty$。然而,我很难构建这样的递归关系,我发现自己陷入了困境</p>(第页) https://math.stackexchange.com/q/4933843 2 高斯过程的运行最大值分布有闭合形式的解吗? 大安 https://math.stackexchange.com/users/881138 2024-06-17时间16:18:22 2024-06-17T16:18:22Z <p>设$X=(X_t)_{t\geq0}$</span>是方差为零均值的高斯过程{E} X _ t X _ t^\顶部$</span>并定义<span class=“math-container”>$$S_T=\sup_{T\leq T}X_T\quad T\in[0,\infty)$$</span>$X$</span>的运行最大值</p>(第页)<p><strong>问题:</strong>如果我们知道$\sigma(T)$</span>中的所有$T\in[0,T]$</sspan>,是否存在$S_T$的封闭形式解决方案</p>(第页)<p>首先,我试图用随机微积分的结果来研究这个问题。也就是说,如果我没记错的话,因为$X$</span>是高斯的,那么对于任何<span class=“math-container”>$T$</sspan>,停止的进程是鞅,如果$\sigma(T)$是可预测的,那么根据鞅表示定理,我们可以写<span class=“数学容器”>$$X_T=\int_0^T\xi\mathrm{d} W公司\四边形{P}(W)_T$$</span>其中,$\mathcal{P}(W)_T$</span>表示渐进过程的空间,其中$\mathbb{E}\int_0^TX_T^2\mathrm{d}\langle W\rangle_T=\mathbb{E}\ int_0^ TX_T ^2\ mathrm{d} t吨&它;\infty$</span>。因此,<span class=“math-container”>$X$</span>可以写成关于布朗运动的随机积分,并且布朗运动的运行最大值已经得到了很好的研究。即,对于布朗运动,我们知道<span class=“math-container”>$$P(\sup_{t\leq t}W_t&gt;x)=P(|W_t|&gt;x)\quad\对于所有x\in\mathbb{R},\quad_for所有t\in[0,\infty)$$</span>因此<span class=“math-container”>$\sup_{t\leq-t}W_t\sim|W_t|$</span>。现在直觉告诉我也许<span class=“math-container”>$$S_T\sim|N(0,\int_0^T\sigma(T)\mathrm{d} t吨)|$$</span>假设这与布朗运动的情形一致,即$\int_0^T\mathrm{d} t吨=T$</span>,但提出证据比我预期的要难</p>(第页)<p><em>备注:作为一个小备注,我希望使用结果来判断一个过程是否确实是高斯过程。也就是说,在某些假设下,过程$X$</span>应该是高斯的,而在另一种假设下,对于每个<span class=“math-container”>$t&gt;0$</span>,因此,如果我们知道所有<span class=“math container”>$t\in[0,t]$</span>的<span class=“math container”>$\sigma(t)$</span>,则可以<em>统计测试</em>,在<span class=“math container”>$t$</span>时,我们观察到<span class=“math container”>$S_T$大于某个值$x\in\mathbb{R}$</p>(第页) https://math.stackexchange.com/q/2333938 2 平方正态随机变量的MGF 苏珊_马特123 https://math.stackexchange.com/users/272910 2017年6月23日18:09:14Z 2024-06-17T12:28:05Z <区块报价><p>如果$X\sim\mathcal{N}(\mu,\sigma^2)$和$Y=X^2$,那么$Y$的力矩生成函数(MGF)是什么</p>(第页)</blockquote><p><strong>回答:</strong>Y的MGF是:</p><p>$$\mathbb{E}\left[E^{tY}\right]$$</p><p>但是,首先我必须计算$Y$的PDF:$$\mathbb{P}(Y\leqx)=1-\mathbb{P}(x^2\geqx)=1-2\mathbb}(x\geq\sqrt{x})$$我可以继续,但路还很长。有更短的路吗</p>(第页)<p>如果$X$是标准正态分布($\mu=0$,$\sigma^2=1$),则$Y$将是卡方分布,其MGF对于$t&lt;为(1-2t)^{-0.5};0.5$. 我怎样才能用它来获得我需要的东西</p>(第页)